Find the perimeter
...................

Find The Perimeter...................

Answers

Answer 1

Answer:

58 inches

Step-by-step explanation:

15+12=27+14=41+9=50+8=58


Related Questions

please answer this question​

Answers

Given that the variables are complex numbers, the value of the complex expression [tex]\frac{\gamma}{\alpha} +\bar{\frac{\alpha}{\beta}}[/tex] is -2

How to solve the complex expression?

The given parameters are:

[tex]\alpha + \beta + \gamma = 0[/tex]

[tex]\frac{1}{\alpha} + \frac{1}{\beta} + \frac{1}{\gamma} = 0[/tex]

Rewrite the first equation as:

[tex]\alpha + \beta = - \gamma[/tex]

Take LCM in the second equation

[tex]\frac{\alpha + \beta}{\alpha \beta} + \frac{1}{\gamma} = 0[/tex]

So, we have:

[tex]\frac{ - \gamma}{\alpha \beta} + \frac{1}{\gamma} = 0[/tex]

Rewrite as:

[tex]\frac{1}{\gamma} = \frac{\gamma}{\alpha \beta}[/tex]

Multiply through by [tex]\alpha[/tex]

[tex]\frac{\alpha}{\gamma} = \frac{\gamma}{\beta}[/tex]

Inverse both sides

[tex]\frac{\gamma}{\alpha} = \frac{\beta}{\gamma}[/tex]

Make [tex]\beta[/tex] the subject in [tex]\alpha + \beta + \gamma = 0[/tex]

[tex]\beta =-(\alpha + \gamma)[/tex]

So, we have:

[tex]\frac{\gamma}{\alpha} = \frac{-(\alpha + \gamma)}{\gamma}[/tex]

Expand

[tex]\frac{\gamma}{\alpha} = -\frac{\alpha}{\gamma}- 1[/tex]

This gives

[tex]\frac{\gamma}{\alpha} +\frac{\alpha}{\gamma} = - 1[/tex]

Make [tex]\gamma[/tex] the subject in [tex]\alpha + \beta + \gamma = 0[/tex]

[tex]\gamma = -(\beta + \alpha)[/tex]

So, we have:

[tex]\frac{\gamma}{\alpha} +\frac{\alpha}{-(\beta + \alpha)} = - 1[/tex]

Split

[tex]\frac{\gamma}{\alpha} +\bar{\frac{\alpha}{\beta}} + \frac{\alpha}{\alpha} = - 1[/tex]

Evaluate the quotient

[tex]\frac{\gamma}{\alpha} +\bar{\frac{\alpha}{\beta}} + 1 = - 1[/tex]

Subtract 1 from both sides

[tex]\frac{\gamma}{\alpha} +\bar{\frac{\alpha}{\beta}} = - 2[/tex]

Hence, the value of [tex]\frac{\gamma}{\alpha} +\bar{\frac{\alpha}{\beta}}[/tex] is -2

Read more about complex numbers at:

https://brainly.com/question/10662770

#SPJ1

What is the approximate volume of a cylinder with a radius of 3 centimeters and a height of 4 centimeters?

Use 3.14 for π and round the answer to the nearest tenth.

Answers

Answer:

113.04 cm

Step-by-step explanation:

Answer:113

Step-by-step explanation:

The area of the pool when viewed from above is approximately 78.5 m2. What is the radius of the pool? (Use 3.14 for pi.)


5 m


11 m


6 m


12 m

Answers

Step-by-step explanation:

I guess it is a circular pool.

the area of a circle is

pi×r²

in our case we know

pi×r² = 78.5

3.14×r² = 78.5

r² = 78.5/3.14 = 25

r = 5 m

Find the distance between point P and line I
Line I contains points (3,2) and (7,-1). Point P has coordinated (2,9)

Answers

Answer: 8

Step-by-step explanation: divion

Answer:

Step-by-step explanation:

Determine the linear function of the line that goes through the points (2; 6) and
(6; 5).
(1) = −6,50 + 0,50;
(2) = 0,25 − 6,50;
(3) = 6,50 − 0,25;
(4) = −0,65 − 0,25x

Answers

Answer:

[tex]y=6.50-0.25x[/tex]

Step-by-step explanation:

[tex]\textsf{let}\:(x_1,y_1)=(2,6)[/tex]

[tex]\textsf{let}\:(x_2,y_2)=(6,5)[/tex]

[tex]\textsf{slope}\:(m)=\dfrac{y_2-y_1}{x_2-x_1}=\dfrac{5-6}{6-2}=-0.25[/tex]

Point-slope form of linear equation:  [tex]y-y_1=m(x-x_1)[/tex]

(where m is the slope and (x₁, y₁) is a point on the line)

Substituting the found slope and a point on the line:

[tex]\implies y-6=-0.25(x-2)[/tex]

[tex]\implies y-6=-0.25x+0.5[/tex]

[tex]\implies y=-0.25x+6.5[/tex]

Rearranging the equation to match the answer options given:

[tex]\implies y=6.50-0.25x[/tex]

Help me solve equations
2=2+g/4

Answers

Answer:

g = 0.

Step-by-step explanation:

Solve:

2 = 2 + g/4g/4 = 2 - 2= g/4 = 0.g = 0.Answer is g = 0.

       [tex]\rule{250}{2}[/tex]

[tex]\dashrightarrow\large\blue\textsf{\textbf{\underline{Given question:-}}}[/tex]

    Solve 2=2+g/4

[tex]\dashrightarrow\large\blue\textsf{\textbf{\underline{Answer and how to solve:-}}}[/tex]

When solving an equation, we need to make sure all variables are on the left-hand side, and all numbers are on the right.

Since we have numbers on both sides, we need to move them to one side, as follows:-

[tex]\Longrightarrow\sf{2-2=g/4}[/tex]

[tex]\longrightarrow\sf{0=g/4}[/tex]

Now, we multiply by 4 on both sides to get rid of the fraction:-

[tex]\longrightarrow\sf{0\times4=g/4\times4}[/tex]

[tex]\longrightarrow\sf{0=g}[/tex]

So we conclude that g=0.

Good luck with your studies.

[tex]\rule{300}{3}[/tex]


What is the value of the x-coordinate of point A?

Choose 1 answer

A) sin(50)
B) cos(50)
C) sin(140)
D) cos(140)
E) sin(230)
F)cos(230)

Answers

Answer: F (cos(230)).
Explanation:
Cosine corresponds to the x coordinate of points on a circle when the radius is drawn in like this, so the answer would be the cos of that angle, but you have to account for the fact that the 50 degree angle marked is just the reference angle not the entire angle so you have to add 180 degrees to that to get 230.

PLS PLS HELP HELP ME ITS EASY ITS FREE PONTS LOL PLS PLS HELP HELP ME ITS EASY ITS FREE PONTS LOL PLS PLS HELP HELP ME ITS EASY ITS FREE PONTS LOL PLS PLS HELP HELP ME ITS EASY ITS FREE PONTS LOL PLS PLS HELP HELP ME ITS EASY ITS FREE PONTS LOL PLS PLS HELP HELP ME ITS EASY ITS FREE PONTS LOL PLS PLS HELP HELP ME ITS EASY ITS FREE PONTS LOL PLS PLS HELP HELP ME ITS EASY ITS FREE PONTS LOL

Answers

Answer:

42 degrees

Step-by-step explanation:

48 + x = right angle = 90 degrees

48+ x = 90

x = 42 degrees

NO LINKS!!! Please help me with this graph part 3a​

Answers

Answer:

g(x) = -|x| - 8

Step-by-step explanation:

Finding the values of a, h, and k :

a = slope of the line

a = -10 + 8 / 2 - 0a = -2/2a = -1

h = any horizontal shift (left/right)

As it lies on the y-axis, we can conclude no horizontal shift has taken placeh = 0

k = vertical shift (up/down)

It has shifted 8 units downk = -8

Forming the equation :

⇒ g(x) = (-1)|x - 0| - 8

g(x) = -|x| - 8

Answer:

[tex]\large{\boxed{\sf y = -1|x-0| -8}}[/tex]

Explanation:

Absolute value of a graph formula:

y = a |x -h| + k

Identify the vertex : (h, k) = (0, -8)

Take two points: (0, -8), (1, -9)

[tex]\sf Find \ slope \ (a) : \sf \ \dfrac{y_2 - y_1}{x_2- x_1} \ = \ \ \dfrac{-9-(-8)}{1-0} \ \ = \ \ -1[/tex]

Put them together :  [tex]\bf y = -1|x-0| -8[/tex]

When Maddy was born, her aunt opened a certificate of deposit in her
honor to help send her to college some day. Now at the age of 18, there is
$39,322 in Maddy's account. How much did her aunt originally invest if
the interest rate was 5.5%? Round to the nearest dollar.

Answers

Given the final amount of money available in Maddy's account, the interest rate and the time elapsed, the initial money her aunt invested was $15,000.

What is an interest in banking?

Interest is simply the amount of money a lender or financial institution receives for lending out money or pays for receiving money.

The formular for calculating compound interest is expressed as;

A = P(1 + r/n)^(n*t)

Where A is final amount, P is initial principal balance, r is interest rate, n is  number of times interest applied per time period and t is number of time periods elapsed.

Given the data in the question;

Interest rate r = 5.5% anuually = 5.5/100 = 0.055Final amount A = $39,322 Time r = 18 yrsInitial principal balance P = ?

We substitute our given values into the expression above.

$39322  = P(1 + 0.055/1)^(1*18)

$39322  = P(1 + 0.055)^(18)

$39322  = P(1.055)^(18)

P = $39322 / (1.055)^(18)

P = $39322 / 2.621466

P = $15,000

Therefore, given the final amount of money available in Maddy's account, the interest rate and the time elapsed, the initial money her aunt invested was $15,000.

Learn more about compound interest here: brainly.com/question/27128740

#SPJ1

Identify the vertex, equation of the axis of symmetry, y-intercept, and zeros of the graph of a quadratic function.

Answers

Answer:

See below ~

Step-by-step explanation:

Details of Graph

Vertex = (-3, 4)Axis of Symmetry : x = -3Y-intercept : -5Zeros : (-5, 0) and (-1, 0)

The measure of angle 1 (10x+8) and the measure of angle 3 (12x-10). What is the measure of angle 2 in degrees? 9 98 82 16

Answers

Answer: 82

Step-by-step explanation:

Question 14 of 15
What are the domain and range of the inequality ?

Answers

Answer:

we see that x must be greater then 3 and y which is the domain must be greater then 5 for the inequality to be true.

so option D is the correct option.

let me know if you need any more help!

Let's Check In
Identify the slope of the following equation.
1
4x+y=3
A. 72 = -2
B. m = 4
C = -8
D = 10

Answers

Answer:

slope (m) = -4

Step-by-step explanation:

Equation: 4x + y = 3

need to get into to y = mx + b form

m is the slope

b is the y-intercept

4x + y = 3         subtract 4x from both sides

4x - 4x + y = 3 - 4x

y = -4x - 3

m = -4

A. 72 = -2       not true expression

B. m = 4          positive 4 not negative    

C = -8             ?????

D = 10             ?????

none of the choices are right

Intelligence Quotient (IQ) scores are often reported to be normally distributed with μ=100.0 and σ=15.0. A random sample of 43 people is taken. Step 1 of 2 : What is the probability of a random person on the street having an IQ score of less than 98? Round your answer to 4 decimal places, if necessary.

Answers

Using the normal distribution, it is found that there is a 0.4483 = 44.83% probability of a random person on the street having an IQ score of less than 98.

What is probability?

Probability is defined as the ratio of the number of favourable outcomes to the total number of outcomes in other words the probability is the number that shows the happening of the event.

Normal Probability Distribution

In a normal distribution with mean  and standard deviation, the z-score of a measure X is given by:

[tex]Z=\dfrac{x-\mu}{\sigma}[/tex]

It measures how many standard deviations the measure is from the mean.After finding the z-score, we look at the z-score table and find the p-value associated with this z-score, which is the percentile of X.

In this problem, the mean and the standard deviation are, respectively, given by and.

The probability of a random person on the street having an IQ score of less than 98 is the p-value of Z when X = 98, hence:

[tex]Z=\dfrac{x-\mu}{\sigma}[/tex]

[tex]Z=\dfrac{98-100}{15}[/tex]

[tex]Z=-0.13[/tex]

Has a p-value of 0.4483.

0.4483 = 44.83% probability of a random person on the street having an IQ score of less than 98.

To know more about probability follow

brainly.com/question/24663213

#SPJ1

a bookstore sells 7 books for $99.75 which table represents the relationship between number of books and total price?

Answers

Answer:

You did not provide the tables to choose from, but the relationship between number of books and total price is that each book is 14.25. Hope that helps at least.

When a plane intersects a sphere, what two-dimensional shape describes the cross-section? ​

Answers

Answer:

A Circle

Step-by-step explanation:

Calculate the accumulated amount after 3 years, if R5 000 is invested at an interest rate of 10% compounded quarterly?​

Answers

Answer:

R6,724.44 or so I think that's right

[tex]~~~~~~ \textit{Compound Interest Earned Amount} \\\\ A=P\left(1+\frac{r}{n}\right)^{nt} \quad \begin{cases} A=\textit{accumulated amount}\\ P=\textit{original amount deposited}\dotfill &\$5000\\ r=rate\to 10\%\to \frac{10}{100}\dotfill &0.10\\ n= \begin{array}{llll} \textit{times it compounds per year}\\ \textit{quarterly, thus four} \end{array}\dotfill &4\\ t=years\dotfill &3 \end{cases} \\\\\\ A=5000\left(1+\frac{0.10}{4}\right)^{4\cdot 3}\implies A=5000(1.025)^{12}\implies A\approx 6724.44[/tex]

A high school has 44 players on the football team. The summary of the players' weights is given in the box plot. Approximately, what is the percentage of players
weighing greater than or equal to 167 pounds?

Answers

The percentage of players weighing greater than or equal to 167 pounds is 75%.

What percentage of students weigh greater than 167 pounds?

A box plot is used to study the distribution and level of a set of scores. On the box, the first line to the left represents the lower (first) quartile. 25% of the score represents the lower quartile.

On the box plot, the lower quartile is 167.

Students that weigh greater than 167 = 100 - 25 = 75%

To learn more about box plots, please check: https://brainly.com/question/27215146

#SPJ1

Kathy sells encyclopedias and earns $40 for each set she sells. How many sets did she sell if she earned $600 last week?

Answers

Answer:

15

Step-by-step explanation:

Given:

Kathy sells encyclopedias and earns $40 for each set she sells.

To Find:

How many sets did she sell if she earned $600 last week?

Solve:

Since kathy sells encyclopedias and earns $40 for each set she sells.

And if kathy total is $600

then we do

$600 divide by $40

600/40 = 15

Hence, she sold 15 sets last week.

Kavinsky

Answer:

15

Step-by-step explanation:

This is basically 600/40

600/40 = 15

Please help !!! 50 points!!!!!!

Answers

Step-by-step explanation:

I could not see the entire question

if the question was 6√5²×6√5³×6√5⁴

the answer would be 5√5

if the question was 6√5²×6√5³×6√5

without exponent 4

5²×5³×5=15625

6√15625

the answer will be 5

Which survey question could have been asked to produce this data display?

A. How long did it take you to walk to the waterfall and back?
B. How many times did you walk to the park last month?
C. How far did you walk today?
D. What is the length of the street you live on?

Answers

Answer:

C

Step-by-step explanation:

The only way to know if you he waked this amount of distance is to know how much they walked today! So the answer is C

Answer: Your answer is C how far did you walk today?

Step-by-step explanation: I did the k12 unit test. (For proof) it has 10 questions. Question number sevens question is Which term describes the distribution of this graph? and part of the name is Unit Test: Data Distribution - Part 1.

Hope it helped please mark me brainliest :D

please help and ty if u do <33​

Answers

Answer:

[tex]3^{-5} =\frac{1}{3^{5} } = (\frac{1}{3} )^{5}[/tex]
[tex]5^{-3} = \frac{1}{5} *\frac{1}{5} *\frac{1}{5} = \frac{1}{5*5*5}[/tex]

Step-by-step explanation:

15 and 1/15 do not equal anything.

Hope this helped!

Solve the division problem:
10)430
O A. 40
B. 43
C. 400
D. 403

Answers

Answer is option B.43

430÷ 10 = 43

By : Modern Einstein

Please help!! Will give brainliest if it’s the correct answer

Maggie wants to buy a car for $24,500. If she puts a down payment of $4,000 down, she will have to take out a loan for the rest of the money with an interest rate of 5.1% and pay it back over 6 years (72 months). How much will Maggie pay each month on her loan?


$299.24

$284.79

$340.27

Answers

Answer:

It's A I think

Step-by-step explanation:

hope this helps

Answer:

299.24

Step-by-step explanation:

take away the down payment from the total. multiply the interest rate then add it to the total. Then divide it by 6 to see how much she pays a year. then divide by 12 for monthly payments

please help me thank you​

Answers

Answer:

2.5

Step-by-step explanation:

mean=sumfx/sumf

1×2+2×2+3×5+4×1/2+2+5+1

25/10

2.5

68.8% complete
Question
You work for a lawn service and need to prepare fertilizer for the day. The directions state that you should mix four ounces of fertilizer for every ten gallons of water. The tank holds 50 gallons of water. You want to have a full tank to start the day’s work. How many ounces of fertilizer should you add to the tank?

A.4 ounces
B.8 ounces
C.20 ounces
D.40 ounces
E.200 ounces

Answers

Answer: 20. D.

Step-by-step explanation: 10 times 5 = 50. 4 times 5 = 20.


I hope this is right.

The answer for the problem is c. 20 ounces

If p varies jointly as q and the cube
of r, and p
= 270 when q = 6 and
r = 3, find p when q = 3 and r = 5.

Answers

Answer:

625

Step-by-step explanation:

p= Kqr³

where K is the constant

K = p/qr³ = 270/6•3³

K= 5/3

p = 5/3• qr³

p = 5/3• 3• 5³

p = 5⁴

p= 625

Hope it helps! :)

What kinds of sports are played by one person?
A. Mixed
B. Individual
C. none of these
D. Team

Answers

the answer for this is b

Henry made a shirt using 1/3 yards of yellow fabric and 1/2 yards of red fabric. How many yards of fabric did he use in all?
Write your answer as a fraction in simplest form.

Answers

I think that the answer is 123

Other Questions
100 POINTSPlutonium-235Given: energy released = about 200 MeV per individual reaction (mass = 239 amu)Part AList the balanced nuclear reaction.I NEED THIS ASAP x2 + 5x + 6x2 + 8x + 12y2-7y + 12y? - 11y + 28x2 - 3x - 40 Optimum engine oil pressure at operating temperature and moderate engine load should be __________ ps List all the words which are pluralized by simply adding s to the f. Josh calculated the mean absolute deviation of the points he earned throughout the year in four of his classes: science, math, Spanish, and art.science: MAD of 1.14math: MAD of 1.6Spanish: MAD of 1.2art: MAD of 1.93In which class were his scores the closest together?scienceartSpanishmath HELPPPP!!!! What is the difference between buen amigo and amigo bueno speech on what does succes mean to you A screen print titled marilyn by andy warhol. colored multiple portraits of marilyn monroe are on the left. right is a black and white version. what format did warhol borrow to create the above images of marilyn monroe? a. printing press style format used for newspapers b. diptych format from the icons of christianity such as those found in the byzantine church c. woodcut style used in germany giving brainliest!!!! Try This question out and Ill give you brainliest no links or I will report you what is the main function of the red cells The garden shown at right is divided into two section by a fence. If the quadrilateral section is a square, what is the square area? (6ft : 8ft) I'm taking a test and don't know how to solve this problem :') Use what you know about triangles to classify this triangle.Triangle with two acute angles and one obtuse angle. Two sides are equal in length.Group of answer choices A.point 1B.point 2C.point 3D.point 4E.point 5 what were some of the populist paths taken following the Great Depression? How to find the surface area of a triangular prism Does anybody know this?Find the area of the shaded region if PA=7 inches and PB=4 inches 2. What are the x-intercepts of the graph of the function defined by (x - 2)(2x + 1)? A. (2,0) and (-1,0) B. (2,0) and (1,0) C. (-2,0) and (1,0) D. (-2,0) and (-1,0) How many minutes did Todd use in October